Kvantummechanika Lagrange-formalizmus alapján??

Örökmozgók, 100% feletti hatásfok
Avatar
szabiku
Hozzászólások: 943
Csatlakozott: 2016.12.22. 01:27

Kvantummechanika Lagrange-formalizmus alapján??

Hozzászólás Szerző: szabiku » 2017.07.05. 22:31

Kinek mi a véleménye az alábbi cikk gondolatmenetéről?

http://epa.oszk.hu/02400/02498/00005/pd ... 19-024.pdf

Helyes, vagy nem helyes? És miért?
0 x

Avatar
szabiku
Hozzászólások: 943
Csatlakozott: 2016.12.22. 01:27

Kvantummechanika Lagrange-formalizmus alapján??

Hozzászólás Szerző: szabiku » 2017.08.11. 19:41

A hatásintegrál szempontjából a kvantumfizikai Lagrange-sűrűség tulajdonképpen skalárszorzat magját képezi. Komplex függvények skalárszorzatáról van szó. Ennél pedig a függvényekre ható operátorok átháríthatók a szorzat másik tényezőjére úgy, hogy közben az adjungáltjukat kell venni. Az ide vonatkozókat (5), (6), (8) tartalmazza is. A négy Lagrange-sűrűség csupán ilyen átalakításoknak megfelelően különbözik egymástól.

- A (14) alatti 1.-es kérdés megkérdőjelezi a Lagrange-sűrűségek egyenértékűségét, miután (12) alatt ki van jelentve, hogy a variációs eljárás során mindegyik a Schrödinger-egyenletet adja.
Ez egyébként valóban mély és lényegi gondolat, olykor én is agyalok rajta, de az is ott van ez mellett ellenben, hogy a szimmetriákat is az adja, ha valamilyen átalakítása a hatás variációját nem befolyásolja.

- A 2. azt állítja, hogy a Lagrange-formalizmus egyértelmű.
Az én meglátásom szerint a klasszikus PONTmechanika Lagrange-formalizmusa félrészt nem működik a kvantumfizika területén. Az említett operátoráthárításokat nem tudja követni, mert a Lagrange-sűrűség csupán a térfogati integrál magja, és az csak integrálostól lenne az eredeti klasszikus elmélet szerinti Lagrange-függvény. A Lagrange-sűrűség becsapja a formalizmust, így azt itt részben kidobhatjuk. (A folytonos anyageloszlással dolgozó relativitáselméletben is elmarad ezen formalizmus jó része..) Ha a Lagrange-sűrűséget önmagában nézzük (tehát az integrál magját), akkor egymáshoz viszonyítva valóban nem egyenértékűek a (9)-(12) felírások, de a hatás variációja szempontjából egyenértékűek. A (15) és (16) kanonikusan konjugált "impulzus" csak a Lagrange-sűrűség bizonyos "szimmetrikus" alakja esetén értelmes és ellentmondásmentes. Ebben a nem relativisztikus esetben nem jó, értelmetlen és hibás (pl. jobboldalaik nem egymásnak konjugáltjai..). A speciálisan relativisztikus kvantumtérelmélet esetén viszont már van a Lagrange-sűrűségnek olyan megfelelő (szimmetrikus) alakja, amely esetén értelmesen felírható a kanonikusan konjugált "impulzus", és akkor (17) is jó lesz (Hamilton-sűrűség).

(Mellesleg (18) szerintem [H,F] http://www.tankonyvtar.hu/hu/tartalom/t ... x1-14004r2 (19)-nél hiányzik az F esetleges explicit időfüggése miatti parciális időderivált a jobb oldalon.. Valamint nem is értem, hogyan jöttek ki a (24)-(31) eredmények, hiszen a (15)-(23) képletek egyáltalán nem ide valóak, ezért hibásak is: pl. (15),(16); (20),(22); (21),(23) jobboldalaik nem egymásnak konjugáltjai..)
0 x

Avatar
szabiku
Hozzászólások: 943
Csatlakozott: 2016.12.22. 01:27

Kvantummechanika Lagrange-formalizmus alapján??

Hozzászólás Szerző: szabiku » 2018.10.02. 05:18

G.Á írta:
2018.09.28. 10:45
Mi a kérdésed?
szabiku írta:Szerinted rendben van a 16.2 fejezeti propagátor?
G.Á írta:Nem látok lényeges hibát.
Én pedig csak azt látok. :D

https://www.docdroid.net/esgwpBm/quantu ... mateur.pdf 146. oldal:

"The real beauty of a Green’s function is the property that we had in eqn 16.6, namely that (16.13)."
>A Green-függvény valódi szépsége az a tulajdonság, amelyet az x(t) = ∫du G(t,u)f(u) egyenlet mutat, nevezetesen (16.13).

(16.13) φ(x,tx) = ∫dy G+(x,tx,y,ty)φ(y,ty) .

#Honnan vette az f(u) --> φ(y,ty) megfeleltetést?

"the Green’s function propagates the particle from the spacetime point (y,ty) to (x,tx)"
>A Green-függvény terjeszti a részecskét a téridő (y,ty) helyéről az (x,tx) helyére.

#Szerintem a propagátor nem ilyen értelemben lett elnevezve, hanem, hogy a klasszikus korpuszkuláris részecskekép helyett egy térben (és időben) elterjesztett "részecske"képet szolgáltat. Azaz a propagátor nem viszi a részecskét, hanem (ki)terjeszti, (el)terjeszti. Tehát ilyen értelemben propagál a propagátor-függvény. A könyvíró félreérti a fogalmat, és kitalált valami hülyeséget.

147. oldal: "the propagator G+(x,tx,y,ty) is the probability amplitude that a particle in state |y> at time ty, ends up in a state |x> at time tx. The interpretation means that the Green’s function may be written (16.18)"
>A G+(x,tx,y,ty) propagátor annak a valószínűségi amplitúdója, hogy a (korábbi) ty időben |y> állapotban lévő részecske, a (későbbi) tx időpontban az |x> állapotba kerül végül. Az értelmezés azt jelenti, hogy a (kauzális G+) Green-függvényre írható:

(16.18) G+(x,tx,y,ty) = θ(tx−ty)<x(tx)|y(ty)> .

(16.19) G+(x,tx,y,ty) = θ(tx−ty)<x|U(tx-ty)|y> .

#Honnan veszi ezeket?? Ez egy kitalált marhaság. Ha egyszerű időfejlődésről van szó, akkor |x> = U(tx-ty)|y> , ami azt jelenti, hogy a skalár szorzatuk 1. Ha pedig |x> és |y> is bázisállapotot jelent, akkor egyszerűen egy Dirac-delta helykoordináta-térbeli hirtelen szétfolyásáról van szó. Ha a kettő közül csak az egyik jelent bázisállapotot, akkor nem fér össze a két felírás.

φ(x,tx) = <x|φ(t)> = ∫dy G+(x,tx,y,ty)φ(y,ty) .

"Note that, using this language, our old friend the wave function φ(x,tx) = <x|φ(t)> is simply the amplitude that a particle is found at (x,tx) irrespective of where it started."
>Megjegyezzük, hogy ennek a nyelvnek a segítségével a régi barátunk, a φ(x,tx) = <x|φ(t)> hullámfüggvény, egyszerűen azon amplitúdó, amellyel egy részecske található az x helyen tx időpontban függetlenül attól, hogy honnan és mikor indult.

φ(x,tx) = <x|φ(t)> . #Ez most komoly?? Hogy φ=φ , az fix. Másként nem tudom értelmezni a felírást, és ehhez |x> bázisállapotot jelent a helykoordináta-térben.


-----------------------

Próbálja rákeverni az egyszerű nemrelativisztikus alapkvantummechanikára a kvantumtérelmélet unitér S szórásmátrixa alapján történő részecskerendszer-állapotváltozás esetének képét. Abban a propagátorfüggvény a kölcsönhatást jelentő "potenciál"-tagból bukkan elő. Az "Schrödinger-egyenlet", ahol , nem csak a nemrelativisztikus alapkvantummechanika egyenlete, hanem általánosan érvényes és kormányzó főegyenlet az egész kvantumelméletben, azaz a kvantumtérelméletben is! Ez egy nagyon lényeges dolog. (A TUDAT, a "mozgó" külvilág kvantumelméleti állapotának központja vagy lakhelye, alapvetően a számára, azaz a megfigyelő számára nyugvó inerciarendszerben honos. Az elmélet relativisztikusságának ezen "Schrödinger-egyenlet" mögött kell megvalósulnia. Ha a szabad skalárbozon hullámegyenletéről leválasztjuk a relativisztikus nyugalmi energiához tartozó nagyfrekvenciás részt, mint relativisztikus korrekciót, akkor a hasonló alakú igazi eredeti Schrödinger-egyenlethez jutunk, amit ψ-vel szoktak felírni.) Itt másodkvantált állapotot jelent. https://www.tankonyvtar.hu/hu/tartalom/ ... /ch08.html

(Valahol írtam, hogy a szokványos potenciáltól, mint a térben rögzített egyszerű potenciáltértől meg kell szabadulni egy tisztességes, a távolhatást minél inkább mellőző kölcsönhatási elmélet felállításánál. Tehát elképzeléseinkből ki kell űzni azt, és úgy fizikát alkotni. De hozzátettem, hogy az mégsem lehetséges, mert ördögi módon valamilyen formában mégis marad. Ezzel persze nincs is baj, ha az már nem távolhatásként funkcionál. A kvantumtérelméleti (QFT) kölcsönhatásképben már jól viselkedik, és az itteni (Fock-térben működő) "Schrödinger-egyenletben" a "potenciál"-tag a Hamilton-operátor kölcsönhatási részét jelenti. Ezt szoktuk perturbációnak is nevezni, mert gyakran perturbációs számítási módszert alkalmazunk rá. Ez a "potenciál"-tag már azért viselkedik jól az előbbi értelemben, mert a régi potenciál szerepét benne az alapvető kölcsönhatást relativisztikusan közvetítő, azaz az elméleti felső határsebesség korlátot tisztelő, de lényegileg kvantumelméleti módon kiterjedt részecskemezők töltik be -->(pl. Landau IV könyv (73,11)-ben az A operátorral jelzett fotontér). Rafinált a természet, úgyhogy a pontosabb kiismeréséhez ki kell (kellett) találnunk a modellezésében valami okosat, hogy még mélyebbre szorítsuk a még mindig távolhatást mutató aggasztó problémáját. Hiába relativisztikus formájú az elektrodinamika, ha egyszerűen csak úgy ott van a térben a töltésre ható vektorpotenciál, még ha retardáltnak is vesszük. A kvantumtérelméletes elektrodinamika (QED) már lenyeli ezt a távolhatás problémát, és kevésbé aggasztó. Persze azért a kvantumtérelméletben is még a mélységekben fel lehet kutatni nyomait valamilyen formában, de lehet totálisan teljesen, azaz végtelen mélységekig nincs matematikai lehetőség eltüntetni minden nyomát. Ennek bizonyítása, vagy cáfolása egy érdekes fizikai matematika feladat lehet.) (Még annyit itt az apróbetűben a potenciálhoz, hogy a mértéktérelméletben (GFT) is övé a második főszerep...)

A kvantumtérelmélet szórásmátrixos kölcsönhatásképét vizsgálva felfedezhetjük, hogy az gyönyörű módon visszakanyarodik önmagába. A megelőlegezetten propagátoroknak elnevezett vákuum-várhatóérték függvények a részecskemezőkhöz tartozó Dirac-deltás inhomogén hullámegyenleteket elégítik ki:

Landau IV könyv 353 - 372. oldalak, csúcspont: Elektronpropagátor és Fotonpropagátor. (...Ezt a témát majd innen folytatom... :geek: )
0 x

G.Á
Hozzászólások: 92
Csatlakozott: 2017.06.23. 22:11

Kvantummechanika Lagrange-formalizmus alapján??

Hozzászólás Szerző: G.Á » 2018.10.02. 10:46

#Honnan vette az f(u) --> φ(y,ty) megfeleltetést?
A Green-függvényeknek van egy definíciója a differenciálegyenletek elméletében. Nem megfeleltetés történik a két eset között, de mindkettő a konkrét egyenletnek megfelel.
#Szerintem a propagátor nem ilyen értelemben lett elnevezve (...)
Jó neked.
#Honnan veszi ezeket?? Ez egy kitalált marhaság. Ha egyszerű időfejlődésről van szó, akkor |x> = U(tx-ty)|y> , ami azt jelenti, hogy a skalár szorzatuk 1. Ha pedig |x> és |y> is bázisállapotot jelent, akkor egyszerűen egy Dirac-delta helykoordináta-térbeli hirtelen szétfolyásáról van szó. Ha a kettő közül csak az egyik jelent bázisállapotot, akkor nem fér össze a két felírás.
Félreérted a jelöléseket.
Egyrészről |x> és |y> valóban a koordináta-sajátállapotok. A szétfolyás stimmel.
Másrészről nagyon sokféle állapot(ok rendszere) alkothat bázist, így a bázisállapot nem egyértelmű kifejezés.
#Ez most komoly??
Ja.
Ezt szoktuk perturbációnak is nevezni, mert gyakran perturbációs számítási módszert alkalmazunk rá. (sic)
Esik az eső, mert esernyőt használok.
Persze olvashattad ezt valahol, csak így kissé vicces.
0 x

Avatar
szabiku
Hozzászólások: 943
Csatlakozott: 2016.12.22. 01:27

Kvantummechanika Lagrange-formalizmus alapján??

Hozzászólás Szerző: szabiku » 2018.10.02. 23:33

szabiku írta:Honnan vette az f(u) --> φ(y,ty) megfeleltetést?
G.Á írta:Nem megfeleltetés történik a két eset között, de mindkettő a konkrét egyenletnek megfelel.
Hát pedig írja, hogy szerinte (16.6) alapján van (16.13). Az f függvény pedig az eredeti vizsgálandó differenciálegyenlet (itt Schrödinger-egyenlet) inhomogenitást okozó része. Na de a Schrödinger-egyenlet homogén, abban f(t)=0 , azaz f(u)=0.

szabiku írta:Ha egyszerű időfejlődésről van szó, akkor |x> = U(tx-ty)|y> , ami azt jelenti, hogy a skalár szorzatuk 1.
... skalárszorzatuk . Lemaradt a Dirac-delta jele.
G.Á írta:... a bázisállapot nem egyértelmű kifejezés.
Igen, tudom, ezért írogattam mindig oda, hogy helykoordináta-térben értve.
szabiku írta:(16.18) G+(x,tx,y,ty) = θ(tx−ty)<x(tx)|y(ty)> .

(16.19) G+(x,tx,y,ty) = θ(tx−ty)<x|U(tx-ty)|y> .

Ha pedig |x> és |y> is bázisállapotot jelent, akkor egyszerűen egy Dirac-delta helykoordináta-térbeli hirtelen szétfolyásáról van szó.
G.Á írta:Egyrészről |x> és |y> valóban a koordináta-sajátállapotok. A szétfolyás stimmel.
Nekem is ez tűnne az egyetlen értékelhető jelentésnek. Viszont ehhez |y> , a részecske hullámfüggvénye, és olyan kell legyen, hogy ty időpontban éppen egy Dirac-delta az y koordinátában. És |x> csupán egy (időfüggetlen) bázisállapotot kell jelentsen. (|y> esetén ty időponton kívül (pl. tx -ben) magától |x> helykoordináta-térbeli sajátállapot nem állhat be, legfeljebb egy mérés hatására, ami a hullámfüggvény összeomlása, de most itt ilyenről nincs szó.) (16.18)-ban nem ez látszik. Szóval ez az elképzelés sem OK. a felírásokra.

Egyébként az összes többi felírás is teljesen zavarodott, csak 1-2 matematikai lépés stimmel, de az önmagában nem jelent semmit.

Látszik, hogy totál nem jó az egész...


Na és szerinted hogyan folyik szét egy éppen Dirac-delta helykoordináta-térbeli sajátállapot?
0 x

G.Á
Hozzászólások: 92
Csatlakozott: 2017.06.23. 22:11

Kvantummechanika Lagrange-formalizmus alapján??

Hozzászólás Szerző: G.Á » 2018.10.03. 09:57

Na és szerinted hogyan folyik szét egy éppen Dirac-delta helykoordináta-térbeli sajátállapot?
Úgy, hogy egy időfejlesztő operátor hat rá. Fizikailag nem releváns esetektől kivéve, ezek az állapotok nem energiasajátállapotok, ezért az időfejlődés operátor más, egyszerűen szólva "szétkent" állapotba viszi át. A részletek a konkrét rendszer Hamilton-operátorától függenek.
Egyébként az összes többi felírás is teljesen zavarodott
Kár hogy így látod. Szerintem még ez a könyv a legkövethetőbb.
0 x

Avatar
szabiku
Hozzászólások: 943
Csatlakozott: 2016.12.22. 01:27

Kvantummechanika Lagrange-formalizmus alapján??

Hozzászólás Szerző: szabiku » 2018.10.04. 04:23

szabiku írta:Na és szerinted hogyan folyik szét egy éppen Dirac-delta helykoordináta-térbeli sajátállapot?
G.Á írta:Úgy, hogy egy időfejlesztő operátor hat rá. Fizikailag nem releváns esetektől kivéve, ezek az állapotok nem energiasajátállapotok, ezért az időfejlődés operátor más, egyszerűen szólva "szétkent" állapotba viszi át. A részletek a konkrét rendszer Hamilton-operátorától függenek.
Hmm... A szétkenődés alakjára gondoltam, tehát a részletekre. (Tudod, én ilyen kukackodó vagyok... :) )
Szerintem a Dirac-delta (azaz, hogy a helykoordináta-térbeli |y> a ty pillanatban Dirac-delta) egyértelműen jelenti (Fourier-transzformációval), hogy milyen impulzussajátállapotok szuperpozíciójában van a ty időpillanatban a részecske. Ami ebben a Dirac-deltás esetben folytonosan végtelen impulzussajátállapotokat jelent. Sőt a Fourier-transzformáció azt is elárulja, hogy a szuperpozíció milyen (komplex) együtthatókkal épül fel. Ezek az adatok pedig megszorítják a Hamilton-operátort, vagyis annak lehetséges formáit. Ez esetben teljes mértékben, vagyis nincs külső potenciáltér, és H=p2/2m , valamint az impulzus és energia tekintetében stacionárius esetről van szó. Ebből máris tudjuk az En energiasajátértékeket, és hogy ezek időfüggetlenek. Látszik, hogy itt a konkrét rendszer az, hogy a részecske szabad. Nincsenek függő részletek, viszont éppen ez mégis igen lényeges...

...Gondoltam rákérdezek, mégis mik az elképzeléseid a Dirac-delta helykoordináta-térbeli szétfolyásáról...
szabiku írta:Egyébként az összes többi felírás is teljesen zavarodott
G.Á írta:Kár hogy így látod. Szerintem még ez a könyv a legkövethetőbb.
El tudod nekem magyarázni, hogy ez: <x(tx)|y(ty)> = <x|U(tx-ty)|y> az egyenlőség pontosan hogyan jön? Betűről betűre, indexről indexre, zárójelről zárójelre, részletesen. Annyit értek belőle, hogy U(∆t) az időfejlesztő operátor, és < ..|.. > skalárszorzatot jelent bra-ket jelölésben.

Akkor azt, hogy φ(x,tx) = <x|φ(t)> egyenlőségben miért van x indexszel ellátva a t idő?
Azzal tisztában vagyok, hogy φ(x,t) = <x|φ(t)> = cx(t) = c(x,t) .

Akkor pl. azt, hogy honnan veszi (16.27)-ben, hogy n[φn(x)φn*(y)e-iEn(tx-ty)] = δ(x-y) ?
Olyat ismerek az ortogonális teljes függvényrendszerre, hogy n[φn(x)φn*(y)] = δ(x-y) .
0 x

123
Hozzászólások: 51
Csatlakozott: 2017.02.07. 04:27

Kvantummechanika Lagrange-formalizmus alapján??

Hozzászólás Szerző: 123 » 2018.10.04. 12:24

szabiku írta:
2018.10.04. 04:23
El tudod nekem magyarázni, hogy ez: <x(tx)|y(ty)> = <x|U(tx-ty)|y> az egyenlőség pontosan hogyan jön? Betűről betűre, indexről indexre, zárójelről zárójelre, részletesen. Annyit értek belőle, hogy U(∆t) az időfejlesztő operátor, és < ..|.. > skalárszorzatot jelent bra-ket jelölésben.
Nekem <x|U(-tx+ty)|y> jön ki :?
0 x

G.Á
Hozzászólások: 92
Csatlakozott: 2017.06.23. 22:11

Kvantummechanika Lagrange-formalizmus alapján??

Hozzászólás Szerző: G.Á » 2018.10.04. 13:32

A szétkenődés alakjára gondoltam, tehát a részletekre.
Akkor konkrét Hamilton operátort kellett volna megadnod.
Néhány konkrét Green-függvény mindenesetre meg van adva pl itt: https://en.wikipedia.org/wiki/Green%27s ... _functions
Szerintem a Dirac-delta (azaz, hogy a helykoordináta-térbeli |y> a ty pillanatban Dirac-delta) egyértelműen jelenti (Fourier-transzformációval), hogy milyen impulzussajátállapotok szuperpozíciójában van a ty időpillanatban a részecske.
Ez oké.
Ezek az adatok pedig megszorítják a Hamilton-operátort, vagyis annak lehetséges formáit.
Ez viszont nagyon nem igaz.
...Gondoltam rákérdezek, mégis mik az elképzeléseid a Dirac-delta helykoordináta-térbeli szétfolyásáról...
H=p2/2m esetében ez jól ismert, benne van a legtöbb bevezető könyvben.
El tudod nekem magyarázni, hogy ez: <x(tx)|y(ty)> = <x|U(tx-ty)|y> az egyenlőség pontosan hogyan jön? Betűről betűre, indexről indexre, zárójelről zárójelre, részletesen. Annyit értek belőle, hogy U(∆t) az időfejlesztő operátor, és < ..|.. > skalárszorzatot jelent bra-ket jelölésben.
Ha ennyit értesz, akkor minden triviális. Különböző időkben kiértékelt állapotok belső szorzatát nem egyszerű közvetlenül kiszámolni, az időfejlesztő operátorral írható át azonos időbeli állapotokra, amit viszont sokszor nem indexelnek.
Pont mint ahogyan Heisenberg képben, a különböző időben értelmezett operátorok kommutációs relációi is eltérnek az egyidő-kommutációs relációktól.

A (16.27) kifejezést meg egyben kell nézni.
0 x

Avatar
szabiku
Hozzászólások: 943
Csatlakozott: 2016.12.22. 01:27

Kvantummechanika Lagrange-formalizmus alapján??

Hozzászólás Szerző: szabiku » 2018.10.04. 21:16

123 írta:Nekem <x|U(-tx+ty)|y> jön ki
Biztos azért, mert hozzá a képletet Nagy Károly Kvantummechanika könyvének 327. oldaláról vetted, és nem a 321. oldalról. (198) kitevőjéből hiányzik egy negatív előjel, (161) OK.
0 x

Avatar
szabiku
Hozzászólások: 943
Csatlakozott: 2016.12.22. 01:27

Kvantummechanika Lagrange-formalizmus alapján??

Hozzászólás Szerző: szabiku » 2018.10.07. 23:49

szabiku írta:Ezek az adatok pedig megszorítják a Hamilton-operátort, vagyis annak lehetséges formáit.
G.Á írta:Ez viszont nagyon nem igaz.
Hát pedig szerintem nagyon is.
A Dirac-delta helykoordináta-térbeli bázisállapot pusztán időfejlődéssel (azaz nincs méréses közbeavatkozás) szerintem kizárólag a végtelen szabad mozgás esetén állhat be (olvasd (16.35) után). És csak egyetlen időpillanatban, de egyben (vagyis egyszer) biztosan. (Ez fontos dolog lesz majd a későbbi fejtegetésem során, ha az ígéret szerint folytatom a fentebbi témázásom.)

Megválaszolom, amit kihagytál:
szabiku írta:Akkor azt, hogy φ(x,tx) = <x|φ(t)> egyenlőségben miért van x indexszel ellátva a t idő?
Azzal tisztában vagyok, hogy φ(x,t) = <x|φ(t)> = cx(t) = c(x,t) .
A teret a felmerülő kalkulációk során egy képletformulán belül kétszer is kell koordinátaparaméterezni. Ezt külön betűvel jelöli: x és y. (y helyett ∆x használata vacakabbá tenné a felírásokat.) Viszont az időnél nem ez a helyzet, és csak megkeverő a tx és ty használata t és ∆t helyett. Hasonlóan a térhez kettős paraméterezést használ az időre, de nem akar eltérni a t betűtől, ezért az x vagy y koordinátaparaméterezéshez tartozást indexeléssel jelöli: tx és ty. Ez persze nem csak az amatőr olvasót, hanem az amatőr szerzőt is megkeveri. Nagyon fontos, hogy tx és ty nem egy konkrét időpontot jelölnek az egy t időskálán, mint ahogy azt megszokottan gyakran ilyen indexeléssel jelöljük. A könyv szövegezése pedig pont azt sugallja, teljesen összezavar. Kijavítom a korábbi ilyen hozzászólásom:
szabiku írta:Szerintem a Dirac-delta (azaz, hogy a helykoordináta-térbeli |y> a ty pillanatban Dirac-delta) egyértelműen jelenti (Fourier-transzformációval), hogy milyen impulzussajátállapotok szuperpozíciójában van a ty időpillanatban a részecske.
Szerintem a Dirac-delta (azaz, hogy a helykoordináta-térbeli |y> egy pillanatban Dirac-delta) egyértelműen jelenti (Fourier-transzformációval), hogy milyen impulzussajátállapotok szuperpozíciójában van ebben az időpillanatban a részecske.

Hasonlóan nagyon megtévesztő nemcsak az amatőr olvasónak, hanem az amatőr könyvírónak is, hogy a |φ( .. ,t)> részecskeállapotra (16.18)-ban a kettős paraméterezéshez hasonlóan az |x(tx)> és |y(ty)> jelölésekben az x és y helykoordináta-paraméterek betűjeleit használja, ugyanis ezek után nem tiszta, hogy az |x> és |y> jelölésekkel báziskomponens(eke)t jelöl, vagy részecskeállapotot. Ez a két dolog különböző, kivéve ha a részecskeállapot éppen megegyezik az egyik |x> és |y> báziskomponenssel. (Ez helykoordináta-sajátállapotot vagy helykoordináta-bázisállapotot jelent). (16.13)-ról (16.18)-ra a következő hibás dumával jut:

"the propagator G+(x,tx,y,ty) is the probability amplitude that a particle in state |y> at time ty, ends up in a state |x> at time tx. The interpretation means that the Green’s function may be written (16.18)."
>A G+(x,tx,y,ty) propagátor annak a valószínűségi amplitúdója, hogy a (korábbi) ty időben |y> állapotban lévő részecske, a (későbbi) tx időpontban az |x> állapotba kerül végül. Az értelmezés azt jelenti, hogy a (kauzális G+) Green-függvényre írható (16.18).

Teljesen összezagyválja a dolgokat.

(16.18)-nak és (16.19)-nek szintén köze sincs egymáshoz! (mert (16.13)-nak és (16.19)-nek lesz, csak nem Green-függvény címen... lásd alább.)
szabiku írta:El tudod nekem magyarázni, hogy ez: <x(tx)|y(ty)> = <x|U(tx-ty)|y> az egyenlőség pontosan hogyan jön?
Sehogyan. A két oldal teljesen mást jelent, nem egyeztethetők össze, ezért (16.18) és (16.19) baloldalai sem egyeztethetők össze. Hiba azt mindkettőben G+ -szal jelölni. (16.18) egy későbbi és egy korábbi |φ( .. ,t)> részecskeállapotot skalárszoroz (vetít egymásra), (16.19) pedig az U(∆t) időfejlesztő operátor e-iH∆t helykoordináta-térbeli alakját fejti ki a reprezentációs bázis szerint (aminek egyébként semmi jelentősége vagy haszna nincs..). Ez utóbbi az U(∆t) operátor mátrixreprezentációját jelenti (folytonosan végtelen minden irányban ez a négyzetes mátrix), még az előbbi teljesen mást.

(16.20)-ban az e-iH∆tI operátor főátlóra transzformálatlanságát áthelyezi a mellette álló főátlóra transzformált I identitásoperátorra. Ez a főátlóra transzformálatlanság (az e-ados alak miatt) megegyezik H -éval, így az I átvált |n><n| -re, miközben H átvált a főátlós alakjában szereplő En sajátértékekre. Kell még ehhez egy n összegezés is: e-iH∆tI = n[e-iEn∆t|n><n|] .

Helykoordináta-térben <x|n> az n függvény(ek)re vonatkozó cx = c(x) komplex együttható(k halmaza), ami(ke)t jelölhetünk φn(x) -szel. Mivel <n|x> = (<x|n>)* , ezért <n|y> = φn*(y) . Így jön (16.21) (Az En sajátértékes e-ados kifejezés már csak szám, és független a helykoordinátáktól, így kiemelhető a Dirac-zárójeles integrálos skalárszorzat közepéből.)

És akkor megy tovább a dolog, (16.26)-ban felhasználja a Hφn(x) = Enφn(x) sajátértékegyenletet, és akkor jön a (16.27), amit csalással fejez be, ugyanis n[φn(x)φn*(y)e-iEn(tx-ty)] =/= δ(x-y), mert n[φn(x)φn*(y)] = δ(x-y) , ahogy arra már felhívtam a figyelmet.
G.Á írta:A (16.27) kifejezést meg egyben kell nézni.
Duma. Az igazi Green-függvényes egyenletben a változókhoz tartozó Dirac-delták függetlenek egymástól, ahogyan a változók is! (16.27) végén δ(∆t) és δ(x-y) egymástól független, na de a közvetlen felette álló kifejezés nem ezt mondja. Ez csalás. Egyértelműen hibás. ( --> ** ) Nyilván nem is jöhet ki az, amit az amatőr könyvíró ki akar erőltetni, hiszen már mondtam, hogy (16.19) jobb oldalának semmi köze egyrészt (16.18)-hoz (ami egyébként se nem propagátor, se nem Green-függvény, hanem csupán egy korábbi-későbbi állapotvektor egymásra vett vetülete), másrészt a Green-függvényhez sincs, ahogyan (16.13)-nak sem, hiszen az nem felel meg (16.6)-nak, ahogy azt már fentebb említettem. A (16.19) jobb oldalával képzett (16.13) az OK. , ugyanis az semmi más, csupán egy leképezés: vektor = mátrix szor vektor, azaz φx(t,∆t) = y[G+xy(∆t)φy(t)] csak ez ott a függvényes felírásban (x és y indexek folytonosak). Nézzük (16.13)-at csak az egyszerűség kedvéért elhagyva a + jelölést és Θ(∆t) szorzót:

φ(x,t,∆t) = <x|φ(t,∆t)> = y[<x|U(∆t)|y><y|φ(t)>] = <x|U(∆t)(t)> = φ(x,t+∆t) .

Az U időfejlesztő operátor mátrixreprezentánsát felesleges G-vel jelölni, attól az még nem lesz Green-függvény. Az a helyzet, hogy a könyvben a 16.2 részben sehol sincsen Green-függvény. :D Ez elég nagy égés. A könyv távolabbi részeit nem néztem, de ez és a környező részek gyalázatosan rosszak. Az amatőrök és hozzáértők hülyítése magas fokon.

Igazán szomorú G.Á, hogy ezeket a dolgokat nem veszed észre. Az, hogy api, con, construct vak ilyen tekintetben, nem meglepő. De hogy te is, az már igen.

A propagátorfüggvény lényegi értelmét a relativisztikus kvantumtérelméletben nyeri el. A nemrelativisztikus kvantummechanikában nincs ilyen propagátor. Semmilyen sincs. Annyi van csupán, hogy a részecske nem pontszerű objektum, hanem a térben kiterjedt hullámszerű függvény. A helykoordináta-térbeli Dirac-delta állapot azon az egyetlen időponton kívül, melyben koncentrált, teljesen, azaz végtelenül és egyenletesen szétterült állapotba megy át. Ebben nincs semmi fizikailag jelentős dolog. Ellenben az elméleti felső határsebességet tisztelő relativisztikus kvantumelméletben nincs ilyen azonnali szétkenődés, van jelentősége az efféle propagátorfüggvénynek, ami éppen az inhomogén hullámegyenlet Green-függvénye. A Schrödinger-egyenlet Green-függvénye láthatóan nem az, amit a könyv a 16.2 részben mond.


** --> Folytatom a gondolatmenetet, mert azért érdekes a dolog elemezgetése.
A kauzalitás sem passzol az egészhez, hiszen annak nem kellene a Green-függvényes egyenletből automatikusan következnie, márpedig innen ((16.27)-től) visszafelé tekintve pont az látszik, és azt mondja, hogy a kauzalitás már ezen az alapszinten automatikusan matematikailag benne van az elméletben mindenféle hullámfüggvény összeomlások, csak kifelé futó gömbhullámok, és egyebek nélkül, ami nem igaz. Ha megnézzük a 151. oldalon található 16.4 The many faces of the propagator című részt, akkor a Θ(∆t)-vel (gondolja) kauzálisra vett U időfejlesztő operátor mátrixreprezentánsa (amit hibásan G+ jelöléssel propagátornak és kauzálisnak hisz a két megzavarodott durhami és oxfordi könyvíró) az impulzusreprezentációban, valamint egy idő szerinti Fourier-transzformációval (Example 16.8 , ami teljesen összezavarodott --> ***), némileg hasonló alakú (16.37) a relativisztikus kvantumtérelmélet propagátorfüggvényéhez: . De csak némileg. A pólus vonatkozások kicsit mások, de némileg alternatívok: a kvantumtérelméletben a pólus a "tömeghéjon levéshez", azaz a valódi részecskéhez kapcsolódik, a kommersz kvantummechanikában pedig a rendszer "energia-sajátállapotban levéséhez", azaz a kötött állapotokhoz (stacionárius állapotokhoz) (illetve ennek egy speciális esetéhez:), valamint a teljesen független szabad részecskeállapothoz (ami szintén stacionárius, és egyfajta "kötött" állapotnak tekinthető). Ezt a matematikai közelfekvőséget a Landau IV könyv is említi a 383. oldalon: "A helyzet ugyanaz, mint a nemrelativisztikus kvantummechanikában, ahol is a szórásamplitúdónak pólusa van olyan energiaértékeknél, amelyeknél az ütköző részecskék rendszerével kötött állapotai vannak (III. 128. §)." a szóródó részecskének. (<--ez lemaradt.) Ott azonban egy kicsit másról van szó, nem konkrétan az időfejlesztő operátorról, de az az eset is hasonló, és matematikailag trükkös analitikus dolgokhoz vezet.


*** --> Nem egyszerű ezekből az összevisszaságokból rendesen kielemezni a megalkotójának elborult kvantumelméjét, hogy megértsük sérültségét.
Nézzük előbb magát az Example 16.7 részt:
"Remember the interpretation: we want the amplitude that a particle that starts off in a state |q> will, after a time tx-ty, end up in a state |p>."
>Emlékezzünk az értelmezésre: egy részecske amplitúdójára (hullámfüggvényére) azt akarjuk, hogy az egy |q> állapotban kezdődjön, majd egy tx-ty idő után |p> állapotban végződjön. (hülye egy megfogalmazás..)

Oldalt megjegyzi, hogy <p|q> = δ(p-q) , szóval itt is látható, hogy nem érti a részecske állapota és a reprezentációs bázis közötti alapvető különbséget. Totál amatőr...

(16.35)-ben: <p|U(∆t)|q> = <p|e-iEq(∆t)|q> = <p|q>e-iEq(∆t) = δ(p-q)e-iEq(∆t) .

Pont erről beszéltem a helykoordináta-reprezentációnál (16.2 résznél), hogy ez az egyetlen lehetőség van ilyen (és ennek megfelelő olyan) felírásnál. Teljesen meg van szorítva a Hamilton-operátor, semmi haszna nincs ezeknek a felírásoknak. És ezt ráadásul még be is látja:

"We see that the free particle cannot change its momentum state, so having both p and q is redundant, so we can write the same equation in the following shorthand:"
>Láthatjuk, hogy a szabad részecske nem változtatja meg impulzus állapotát, mindkettőt (p és q) megtartani felesleges, így ugyanazt az egyenletet írhatjuk a következő rövidítésbe: (16.36)-ban az előbbi: e-iEp(∆t) .

Magyarán a diagonális folytonos négyzetes Gpq mátrixát folytonos Gp vektorba zsugorította: Gp = q[Gpq1q] , azaz e-iEp(∆t) = dqδ(p-q)e-iEq(∆t) .

Ezután jön az Example 16.8 rész, ahol Fourier-transzformációval áttér az időtartományról energiatartományra. Úgy tűnik, mintha elengedhetetlen szerepe lenne itt a Θ(∆t) egységugrás függvénynek, de nem. Végezzük el nélküle a transzformációt:

.

Azaz a folytonos Gp vektornak (függvénynek) az energiatartományban a sajátértékeknél szingularitásai (pólusai) vannak. És ennyi. (Ezzel az egyébként nem egyszerű mélységekbe vezető dologgal, nem egy ilyen hibás esetben kellene foglalkozni... de az amatőröknek ez magas...) A többi manipulációnak itt semmi értelme, csak ki akarja hozni azt a képletformát ((16.37) vége), ami némileg hasonlít a fentebb említettre.


----------------------------

A konkrét megtévedés középpontját én a (16.27)-ben elkövetett matematikai hibára tenném (és a (16.37-38) nemidevalóságára), ami egyben egy matematikai közelfekvőséget is mutat. Ez az egész annyira megzavarta a két koma elméjét, hogy a hibák halmára még azzal is rálapátoltak, hogy a kvantumtérelméleti Feynman-gráfos virtuális részecskés többedrendűségben vizsgált belső vonalas dolgokat a 16.3 Turning it around: quantum mechanics from the propagator and a first look at perturbation theory című részben rámagyarázták a kommersz kvantummechanikára. :D
0 x

123
Hozzászólások: 51
Csatlakozott: 2017.02.07. 04:27

Kvantummechanika Lagrange-formalizmus alapján??

Hozzászólás Szerző: 123 » 2018.10.08. 09:36

szabiku írta:
2018.10.04. 21:16
123 írta:
2018.10.04. 12:24
Nekem <x|U(-tx+ty)|y> jön ki :?
Biztos azért, mert hozzá a képletet Nagy Károly Kvantummechanika könyvének 327. oldaláról vetted, és nem a 321. oldalról. (198) kitevőjéből hiányzik egy negatív előjel, (161) OK.
<x(tx)|y(ty)> = <x(0)|y(ty-tx)> mert az időfejlődés unitér, és
<x(0)|y(ty-tx)> = <x(0)| (U(ty-tx)|y>) = <x(0)|U(ty-tx)|y(0)> az U időfejlődés definíciójából
0 x

G.Á
Hozzászólások: 92
Csatlakozott: 2017.06.23. 22:11

Kvantummechanika Lagrange-formalizmus alapján??

Hozzászólás Szerző: G.Á » 2018.10.08. 10:49

(olvasd (16.35) után)
Nézd szabiku, én ezt értem, kb azóta hogy a 2014-15 ös évben levizsgáztam " Többrészecske Green-függvények alkalmazása időfüggő kvantumos transzportfolyamatok leírására" kurzusból.
Pillanatnyilag is lényegében egy ilyen számolást próbálok lefuttatni, noha a cikkben jelenleg egyszer sem szerepel a "propagátor" szó.
És akkor megy tovább a dolog, (16.26)-ban felhasználja a Hφn(x) = Enφn(x) sajátértékegyenletet, és akkor jön a (16.27), amit csalással fejez be, ugyanis ∑n[φn(x)φn*(y)e-iEn(tx-ty)] =/= δ(x-y), mert ∑n[φn(x)φn*(y)] = δ(x-y) , ahogy arra már felhívtam a figyelmet.
*sóhaj*

Egyébként még hálás is lehetnék a félreértéseidért, csak én a gyakoriakat gyűjtöm, te pedig ritkákat adsz.
0 x

Avatar
szabiku
Hozzászólások: 943
Csatlakozott: 2016.12.22. 01:27

Kvantummechanika Lagrange-formalizmus alapján??

Hozzászólás Szerző: szabiku » 2018.10.08. 15:02

G.Á írta:*sóhaj*
én ezt értem, kb azóta hogy a 2014-15 ös évben levizsgáztam " Többrészecske Green-függvények alkalmazása időfüggő kvantumos transzportfolyamatok leírására" kurzusból.
És ott tényleg azt tanították, hogy a n[φn(x)φn*(y)] = δ(x-y) -ba nyugodtan beilleszthetünk az összeg egyes tagjaira egy minden tagra En szerint különböző e-iEn(tx-ty) szorzót, ha az egész kifejezés meg van szorozva δ(tx-ty) -nal??
Ugyan már, ne nevettess! :D :mrgreen:
Hiszen az már úgy nem két Dirac-delta szorzata!
Ekkor a δ(x-y) kifejezés már nem áll, megszűnik, ami úgy nem lesz jó. (van némi matematikai vakságod...)
A Green-függvényes egyenlethez pedig (rendes független) Dirac-delták szorzata kell, hiszen pont az a lényege.
Hogy lehet, hogy nem érted ezt? Nem most doktorálsz? :geek:
0 x

G.Á
Hozzászólások: 92
Csatlakozott: 2017.06.23. 22:11

Kvantummechanika Lagrange-formalizmus alapján??

Hozzászólás Szerző: G.Á » 2018.10.08. 15:49

Amikor azt írom, hogy elemi hibákat vétesz, hasonló dolgokra gondolok.
0 x

Avatar
szabiku
Hozzászólások: 943
Csatlakozott: 2016.12.22. 01:27

Kvantummechanika Lagrange-formalizmus alapján??

Hozzászólás Szerző: szabiku » 2018.10.09. 02:04

123 írta:<x(tx)|y(ty)> = <x(0)|y(ty-tx)> mert az időfejlődés unitér, és
<x(0)|y(ty-tx)> = <x(0)| (U(ty-tx)|y>) = <x(0)|U(ty-tx)|y(0)> az U időfejlődés definíciójából
Igen, jó amit írtál. Köszi. Valóban, igazad van, mert az első felírásodnál ((16.18)-ban) a korábbi állapot vetül a későbbire, tehát az y-os az x-esre. És akkor a szövegezés szerint (ami (16.19)-et ennek megfelelőnek dumálja) a többi felírásnál is a korábbi állapotvektornak kellene a későbbire vetülnie, nem pedig fordítva (ami komplex konjugálásban különbözik). Ha tartjuk magunkat ahhoz, hogy a könyv szerint tx>ty , akkor ha |x> és |y> nem a helykoordináta-reprezentáció bázisát akarja jelenteni (mint ami a felírásokból egyértelmű), hanem (amit a könyvszöveg dumál) inkább részecskeállapotot, akkor az |y> -t nem előre, hanem vissza kellene fejleszteni ∆t -vel, tehát U(-∆t) = U(ty-tx) kell hasson rá. (Valamint mindegy mikor, mert az időfejlődés unitér.) Még ezt is elrontotta a szerző. Ezt észre sem vettem, mert engem jobban zavart, hogy mit is ért pontosan |x> és |y> jelöléseken. Teljesen hibás és zavarodott a könyv, de G.Á professzor érti és jónak véli, ahogyan api.con(struct) is. :mrgreen:
G.Á írta:Egyébként még hálás is lehetnék a félreértéseidért, csak én a gyakoriakat gyűjtöm
:D Írd fel légyszi magadat a fóliádra! THX. xDdd
0 x

G.Á
Hozzászólások: 92
Csatlakozott: 2017.06.23. 22:11

Kvantummechanika Lagrange-formalizmus alapján??

Hozzászólás Szerző: G.Á » 2018.10.09. 11:06

(Valamint mindegy mikor, mert az időfejlődés unitér.) Még ezt is elrontotta a szerző.
Még ezt is elrontottad. Csak időfüggetlen Hamilton-operátornál van így.
0 x

123
Hozzászólások: 51
Csatlakozott: 2017.02.07. 04:27

Kvantummechanika Lagrange-formalizmus alapján??

Hozzászólás Szerző: 123 » 2018.10.09. 11:51

Szóval hogy jön ki <x(tx)|y(ty)> = <x|U(tx-ty)|y>?
0 x

Avatar
szabiku
Hozzászólások: 943
Csatlakozott: 2016.12.22. 01:27

Kvantummechanika Lagrange-formalizmus alapján??

Hozzászólás Szerző: szabiku » 2018.10.09. 22:03

G.Á írta:Csak időfüggetlen Hamilton-operátornál van így.
Csak a zárójeles részt kellett volna idézned, mert amit utána írtam, az nyilván nem erre vonatkozott.
Igen, ezt helyesen mondod, csak ha a Hamilton-operátor nem függ az időtől. (Én is így gondoltam csak ezt elfelejtettem megemlíteni. Szóval ez nem egy elejtett morzsa volt, amit a kiskakas PhD-úr felcsipegethet :mrgreen: -->) És a könyvben éppen ez az esetet áll fenn, hiszen fix En sajátértékekkel kalkulál. (Melléfogtál. :D )
G.Á írta:
2018.09.28. 10:45
Mi a kérdésed?
123 írta:Szóval hogy jön ki <x(tx)|y(ty)> = <x|U(tx-ty)|y>?
G.Á kérlek válaszold meg ezt a kérdést, mert ugye te nagyon érted ezt a 16. fejezetet, el is végezted hozzá a kurzust, és egyet is értesz az összefüggés(ek) helyességével: :D
G.Á írta:Nézd szabiku, én ezt értem, kb azóta hogy a 2014-15 ös évben levizsgáztam " Többrészecske Green-függvények alkalmazása időfüggő kvantumos transzportfolyamatok leírására" kurzusból.
Nézem. (csak nem látom)
G.Á írta:Pillanatnyilag is lényegében egy ilyen számolást próbálok lefuttatni, noha a cikkben jelenleg egyszer sem szerepel a "propagátor" szó.
:shock: :D
Mi a cikk linkje?
Az alapok rendben vannak, amire a téma épül? (Vagy az is olyan bugos, mint ez a kitárgyalt 16. fejezet ebben a hibás amatőr könyvben.)
0 x

con
Hozzászólások: 147
Csatlakozott: 2017.01.13. 12:35

Kvantummechanika Lagrange-formalizmus alapján??

Hozzászólás Szerző: con » 2018.10.09. 22:52

Én is így gondoltam csak ezt elfelejtettem megemlíteni. Szóval ez nem egy elejtett morzsa volt, amit a kiskakas PhD-úr felcsipegethet :mrgreen: -->
Ebben a sorban töményen koncentrálódik mindaz, ami miatt szabiku képtelen kikeveredni a maga körül felépített félreértéshalmazból.
Ha valaki kijavítja, arra ő vagy elkezd kimagyarázkodni, mintha mindig is tudta volna, vagy körömszakadtig védi a védhetetlen, miközben egyre újabb egyéni félreértelmezések sáncát húzza fel az első védelmére.
Amikor érzi mennyire nevetséges, elkezd alpári módon pimaszkodni.
Mindezt sűrűn megszórja a maga kedvenc zöld vigyorkáival.
0 x

con
Hozzászólások: 147
Csatlakozott: 2017.01.13. 12:35

Kvantummechanika Lagrange-formalizmus alapján??

Hozzászólás Szerző: con » 2018.10.09. 23:15

Így megy évek óta, amit időközben sokan eluntak már, ő meg csak dúl-fúl itt magában, mint valami sértett pukkancs. De ha így folytatja, soha se lesz képes megérteni, hogy hol csúszik félre mindig.
0 x

Avatar
szabiku
Hozzászólások: 943
Csatlakozott: 2016.12.22. 01:27

Kvantummechanika Lagrange-formalizmus alapján??

Hozzászólás Szerző: szabiku » 2018.10.10. 01:15

con, te is ezt írtad az indexfórumon a könyv 16. fejezetére:
con írta:Értem persze
Várjuk a pontos választ 123 utóbb feltett kérdésére.
0 x

G.Á
Hozzászólások: 92
Csatlakozott: 2017.06.23. 22:11

Kvantummechanika Lagrange-formalizmus alapján??

Hozzászólás Szerző: G.Á » 2018.10.10. 11:41

Bevallom szabiku, hogy kezdtem azt hinni:
Ha lassan is, de legalább tanulsz, csak éppen az egód körözi le az ismereteidet.

Az utóbbi napokban sajnos fel kellett adnom az ilyen elképzeléseimet.
Ugyanakkor két dolog fennáll, ami miatt nem akarok ebben az álvitában maradni.
1) A QFT tipikusan nem olyan diszciplína, amelyet laikusok igazán képesek elsajátítani, így nem is szép dolog ezért neheztelni.
2) A téma ugyanakkor érzelmileg közelebb áll hozzám, és emiatt talán kevésbé vagyok türelmes veled.
Csak a zárójeles részt kellett volna idézned, mert amit utána írtam, az nyilván nem erre vonatkozott.
Ebben az esetben elnézést, csak (bocsánat a kifejezésért) az idehányt " kicsit más, de némileg alternatív" szövegeid és... képlékeny angol fordításaid között nincs időm kiigazodni.
És a könyvben éppen ez az esetet áll fenn, hiszen fix En sajátértékekkel kalkulál. (Melléfogtál. :D )
Azt hittem általánosságban beszélsz, de ettől függetlenül sajnos az a benyomásom, hogy nem érted mit jelentenek a formulák.
G.Á kérlek válaszold meg ezt a kérdést
Attól függ, hogy mit jelent a "kihozni". A fentihez hasonló ekvivalens alakra hozás az unitér-csoport tulajdonságain alapul.
A véleményem szerint ez azonban nem válaszolja meg azt a kérdést, hogy "honnan jön".
Ha 123-at érdeklik a részletek, javaslom neki a kozmoforum.hu-t. Vagy a szakirodalmat.
Nézem. (csak nem látom)
Látom hogy nem látod. Én meg azt nem látom, hogy a lineáris algebra legalapvetőbb fogalmaival nem voltál tisztában nem egészen két éve.
Kiváncsi vagyok, pótoltad-e?
Mi a cikk linkje?
Az alapok rendben vannak, amire a téma épül?
Még nincs kész.
Azon alapul, hogy egyrészt a kvantumregresszió-tételt mint közelítést, megkerülve számolok többidő-korrelációkat, lényegében a kvantumoptikai P-reprezentációhoz hasonló, de annál jobban használható frame-ben.
0 x

Avatar
szabiku
Hozzászólások: 943
Csatlakozott: 2016.12.22. 01:27

Kvantummechanika Lagrange-formalizmus alapján??

Hozzászólás Szerző: szabiku » 2018.10.11. 00:50

Elfogadom a bocsánat kérésed.

Kár, hogy nincs időd kiigazodni az írásaimon, mert szerintem igen jók, precízek, lényegre törőek, és szépen szerkesztettek is. Sokat lehet belőlük tanulni.

Azért köszönöm a türelmedet. Egy kicsit lehetnél belátóbb, beismerőbb, és akkor nem álvita lenne a diskurzusunk, hanem tisztázás. De sebaj, azt megoldom egyedül is. :geek:
G.Á írta:A QFT tipikusan nem olyan diszciplína, amelyet laikusok igazán képesek elsajátítani
Pedig laikusok voltak azok is, akik később már nem laikusok lettek ebben a tekintetben.
G.Á írta:A téma ugyanakkor érzelmileg közelebb áll hozzám
A fizikát és matematikát, még ha szeretjük is, nem érzelmi alapon kell ténylegesen látni, hanem józan, logikus, érzelemmentes gondolkodással.
G.Á írta:Még nincs kész.
Azon alapul, hogy egyrészt a kvantumregresszió-tételt mint közelítést, megkerülve számolok többidő-korrelációkat, lényegében a kvantumoptikai P-reprezentációhoz hasonló, de annál jobban használható frame-ben.
Jól hangzik, érdekes lehet. Ebben nem vagyok otthon, úgyhogy sok sikert kívánok hozzá. Gondolom a PhD-hez kell.
G.Á írta:Én meg azt nem látom, hogy a lineáris algebra legalapvetőbb fogalmaival nem voltál tisztában nem egészen két éve.
Kiváncsi vagyok, pótoltad-e?
Azért erős az állítás, nem tudom pontosan, mire gondolsz, de nekem is lehetnek hiányosságaim. Igyekszek mindent tudni, ami kell a kutatásaimhoz, megértéseimhez, vizsgálódásaimhoz.
G.Á írta:sajnos az a benyomásom, hogy nem érted mit jelentenek a formulák.
:) :ugeek:
0 x

G.Á
Hozzászólások: 92
Csatlakozott: 2017.06.23. 22:11

Kvantummechanika Lagrange-formalizmus alapján??

Hozzászólás Szerző: G.Á » 2018.10.15. 19:19

Segítségképpen:
A Dirac-delta helykoordináta-térbeli bázisállapot pusztán időfejlődéssel (azaz nincs méréses közbeavatkozás) szerintem kizárólag a végtelen szabad mozgás esetén állhat be.
Ez egyrészt nem igaz, hiszen létezik olyan kvantummechanikai rendszer, amely dinamikájában a hullámfüggvény periodikusan képes tetszőlegesen megközelíteni (idealizált kezdeti állapotban egzaktul eléri) a dirac-delta állapotot. A valóságban persze ilyen nincsen, de mindkettő fenti állítás az idézettel együtt irreleváns.
Nagyon fontos, hogy tx és ty nem egy konkrét időpontot jelölnek az egy t időskálán, mint ahogy azt megszokottan gyakran ilyen indexeléssel jelöljük. A könyv szövegezése pedig pont azt sugallja, teljesen összezavar.
Egyelőre te vagy az első ember az ismeretségi körömben, akit ez a jelölés összezavar. Természetesen két konkrét időpontot jelöl, és ezek természetesen paraméterek.
A G+(x,tx,y,ty) propagátor annak a valószínűségi amplitúdója, hogy a (korábbi) ty időben |y> állapotban lévő részecske, a (későbbi) tx időpontban az |x> állapotba kerül végül. Az értelmezés azt jelenti, hogy a (kauzális G+) Green-függvényre írható (16.18).
Ez igaz.
(folytonosan végtelen minden irányban ez a négyzetes mátrix)
Apróság, de ez így matematikailag nem korrekt.

És akkor megy tovább a dolog, (16.26)-ban felhasználja a Hφn(x) = Enφn(x) sajátértékegyenletet, és akkor jön a (16.27), amit csalással fejez be, ugyanis ∑n[φn(x)φn*(y)e-iEn(tx-ty)] =/= δ(x-y), mert ∑n[φn(x)φn*(y)] = δ(x-y) , ahogy arra már felhívtam a figyelmet.
Duma.
Továbbra is egyben kell nézni a kifejezést, és meglátod hogy nincs csalás.
Az a helyzet, hogy a könyvben a 16.2 részben sehol sincsen Green-függvény. :D Ez elég nagy égés. A könyv távolabbi részeit nem néztem, de ez és a környező részek gyalázatosan rosszak. Az amatőrök és hozzáértők hülyítése magas fokon.
A 16.5-ös példában láthatsz egy Green-függgvényt.

A propagátorfüggvény lényegi értelmét a relativisztikus kvantumtérelméletben nyeri el. A nemrelativisztikus kvantummechanikában nincs ilyen propagátor.
Hülyeség.
Annyi van csupán, hogy a részecske nem pontszerű objektum, hanem a térben kiterjedt hullámszerű függvény. A helykoordináta-térbeli Dirac-delta állapot azon az egyetlen időponton kívül, melyben koncentrált, teljesen, azaz végtelenül és egyenletesen szétterült állapotba megy át. Ebben nincs semmi fizikailag jelentős dolog.

Nem feltétlenül, és általában még csak nem is monoton módon.
*** --> Nem egyszerű ezekből az összevisszaságokból rendesen kielemezni a megalkotójának elborult kvantumelméjét, hogy megértsük sérültségét.
Nézzük előbb magát az Example 16.7 részt:
Nézzük.
"Remember the interpretation: we want the amplitude that a particle that starts off in a state |q> will, after a time tx-ty, end up in a state |p>."
>Emlékezzünk az értelmezésre: egy részecske amplitúdójára (hullámfüggvényére) azt akarjuk, hogy az egy |q> állapotban kezdődjön, majd egy tx-ty idő után |p> állapotban végződjön. (hülye egy megfogalmazás..)
Tényleg hülye fordítás.
Oldalt megjegyzi, hogy <p|q> = δ(p-q) , szóval itt is látható, hogy nem érti a részecske állapota és a reprezentációs bázis közötti alapvető különbséget. Totál amatőr...
Te valamit nagyon-nagyon nem értesz, de magam sem vagyok benne biztos hogy mit. Ez mindenesetre az impulzus-sajátállapotok belső szorzatait jelöli, de ennyit talán felismertél.
Pont erről beszéltem a helykoordináta-reprezentációnál (16.2 résznél), hogy ez az egyetlen lehetőség van ilyen (és ennek megfelelő olyan) felírásnál.
Nem, természetesen bármilyen bázist használhatunk.
Teljesen meg van szorítva a Hamilton-operátor, semmi haszna nincs ezeknek a felírásoknak.
Dőreség.
És ezt ráadásul még be is látja:
"We see that the free particle cannot change its momentum state, so having both p and q is redundant, so we can write the same equation in the following shorthand:"
>Láthatjuk, hogy a szabad részecske nem változtatja meg impulzus állapotát, mindkettőt (p és q) megtartani felesleges, így ugyanazt az egyenletet írhatjuk a következő rövidítésbe: (16.36)-ban az előbbi: e-iEp(∆t) .
Egy nagyon triviális esetet tárgyalt a könyv ezen a ponton.

A konkrét megtévedés középpontját én a (16.27)-ben elkövetett matematikai hibára tenném

Az általad felvetett hiba nem létezik.
(és a (16.37-38) nemidevalóságára),

Didaktikailag ideillik, de persze ez szubjektív.
Ez az egész annyira megzavarta a két koma elméjét, hogy a hibák halmára még azzal is rálapátoltak, hogy a kvantumtérelméleti Feynman-gráfos virtuális részecskés többedrendűségben vizsgált belső vonalas dolgokat a 16.3 Turning it around: quantum mechanics from the propagator and a first look at perturbation theory című részben rámagyarázták a kommersz kvantummechanikára. :D
A Feynman-gráfok a perturbációelmélet egy részét képezik, használhatóak akár a klasszikus fizikában is.
0 x

Avatar
szabiku
Hozzászólások: 943
Csatlakozott: 2016.12.22. 01:27

Kvantummechanika Lagrange-formalizmus alapján??

Hozzászólás Szerző: szabiku » 2018.10.16. 22:57

G.Á írta:A Feynman-gráfok a perturbációelmélet egy részét képezik, használhatóak akár a klasszikus fizikában is.
Hát szerintem az eredeti hamisítatlan Feynman-gráfok konkrétan QED-sek... Innen copyntgatták legfeljebb.
G.Á írta:létezik olyan kvantummechanikai rendszer, amely dinamikájában a hullámfüggvény periodikusan képes tetszőlegesen megközelíteni (idealizált kezdeti állapotban egzaktul eléri) a dirac-delta állapotot. A valóságban persze ilyen nincsen, de mindkettő fenti állítás az idézettel együtt irreleváns.
A Dirac-deltát ahhoz nem lehet eléggé megközelíteni. :) A Dirac-delta csak akkor Dirac-delta, ha az Dirac-delta. Véges módon akár mennyire is közelíted meg, még mindig végtelenül távol vagy tőle, hogy Dirac-delta legyél. Ez a végtelen értéke miatt van így. Kicsit hasonló talán a fény esete is abban a tekintetben, hogy pl. nyugalmi tömeggel rendelkező vektorbozon részecske tetszőlegesen megközelítheti a fénysebességet, de ettől az sosem lesz fény, azaz EM hullám. Szerkezete "végtelenül távol marad" attól, hogy csak két valódi polarizációs állapota legyen, és nem három (a longitudinális polarizációja teljesen valódi marad..).
G.Á írta:Egyelőre te vagy az első ember az ismeretségi körömben, akit ez a jelölés összezavar. Természetesen két konkrét időpontot jelöl, és ezek természetesen paraméterek.
:) Te most kb. azon az állásponton vagy, hogy az f(x) függvényt kifejezetten érdemes konkrétan úgy tekinteni, hogy az inkább egy c érték az xc pontban, mert f(xc) = c, és xc természetesen paraméter.
G.Á írta:Te valamit nagyon-nagyon nem értesz, de magam sem vagyok benne biztos hogy mit. Ez mindenesetre az impulzus-sajátállapotok belső szorzatait jelöli, de ennyit talán felismertél.
Tehát akkor az van, amit írtam: |p> és |q> reprezentációs bázis az impulzustérben. (16.35 és 36) bal oldala G0+(p,tx,q,ty) általános értelmű akar lenni szerintem. Akkor a jobb oldal miért olyan speciális, hogy tetszőleges szuperpozíció részecskeállapot helyett egyszerűen csak sajátállapotok vannak? Ugyan ez a kérdésem (16.19 - 27)-nél.
G.Á írta:Nem, természetesen bármilyen bázist használhatunk.
Mellébeszélsz. Direkt...
G.Á írta:Egy nagyon triviális esetet tárgyalt a könyv ezen a ponton.
Persze. Hol írja a 16.4 és a 16.2, hogy Vigyázat! Csak rettentően nagyon triviális semmire való esetet tárgyalunk. :D Ja igen, ezek csak special Examplék. De a (16.19) előtti mondat nem úgy tűnik. És a 152. oldal tetején miért nem írja, hogy nagyon special Example a very helpful summary??
G.Á írta:Az általad felvetett hiba nem létezik.
Dehogynem. Több sebből is vérzik az egész.
G.Á írta:Továbbra is egyben kell nézni a kifejezést, és meglátod hogy nincs csalás.
Persze. Egyben. Meg nagyon triviálisan.
G.Á írta:Segítségképpen:
<x(tx)|y(ty)> = <x|U(tx-ty)|y> ??

U(tx-ty) vagy U(ty-tx) ??

(16.6) --> (16.13) ??


G.Á! Lehet pont ezek lesznek a beugró kérdések a PhD vizsgán. Meg kell találni a helyes választ! :mrgreen: :mrgreen:
0 x

con
Hozzászólások: 147
Csatlakozott: 2017.01.13. 12:35

Kvantummechanika Lagrange-formalizmus alapján??

Hozzászólás Szerző: con » 2018.10.17. 12:33

Jellegzetes szabikuság. A PhD eljárás és védés menetét, illetve mibenlétét hasonló utánozhatatlan módon érti félre, mint számos fizikai gondolatot és matematikai eljárást.
Azért tanul ennyire nehezen, mert ha az aktuális előismereteivel valamit nem ért meg, akkor rögvest kijelenti, hogy az hülyeség. S mindezt olyan arrogáns módon, hogy mindenkinek elmegy a kedve attól, hogy tanítsa.
Nagyon jellemző volt pár éve, amikor az ELTE bsc képzés kvantumradírral kapcsolatos hallgatói laborkísérletét dorongolta le a az alábbi szakértelemmel:
"Ha hibás a kísérlet, akkor persze meg lehet találni konkrétabban is az elcsúszás okát, de nem olvastam el a szöveget, ugyanis szinte nem is tudok angolul :P , csak annyit szűrtem ki belőle amennyi éppen ehhez a megállapításomhoz kellett.
A kvantumfizikai részéhez még annyira nincs meg a kompetenciám, de lehet megvizsgálom jobban...
Szerintem a rejtett paraméteres probléma is rejtőzhet benne, mert azt is meg lehet csavarni egy kísérlettel úgy, hogy ne vegyük észre annak hibás feltételezését. Ehhez már a képleteket is át kell rágni benne rendesen.
"

"A hibát ott kell keresni, ahol fizikailag nem kielégítő valaminek a megfogalmazása/definiálása, érezhetően hiányos, nem vesz figyelembe valami lényegeset, pongyola, vagy értelmetlen/érthetetlen. A kellő tudással nem rendelkezők ezeken a pontokon simán átsiklanak, és nem veszik észre a becsapást/félrevezetést/ (mert hát ugye hiteles mérési dokumentumként van feltűntetve az egész.) Ezek lehetnek amolyan tudományos tudatmorzsoló vírusok, melyeket mindenfelé terjesztenek, de legfőképp a neten. A média ezeket csak a saját népszerűsítésére használja, és így az a vírusnak dolgozik. :|"

"Hát, megpróbálok valamit még ki okumlálni, de ahhoz nekem is idő kell.. nem mára jön majd össze de holnapra sem az tuti, mert nem vagyok nagy kvantumfizikás. A szakik meg élből nem harapnak jobban erre... ;) (szerintem a fenti okok miatt)
De ha megvan a válasz, kifeszítjük ezt a Walborn gyereket. :)
Vagy részben megdőlnek az eddigi elméletek
:?:"

"Nekem nem tűnik hitelesnek ez a kvantumradír, de megpróbálom értelmezni, a kevés hozzáértésemmel."
"Visszapattan az ernyőről a foton és a becsapódás előtti másik állapotába kerül, mintha a másik résen ment volna keresztül, vagy mi a fene??
:?"
Aztán jön az M, ami hatás nélkül megjelőli a fotont attól függően, hogy melyik résen ment át. Hmm... A rejtett paramétert beállítja, vagy mi a nyavaja?? :| Tovább már nem is elemzem... (két ketvektor félszimbólum összeszorzásával szerintem tenzort tesz egyenlővé vektorral... :? )"

szabiku doktor itt az alábbi kifejezés felett értetlenkedik, vagyis az ő szavajárásával: "elemzi"
|Ψ〉=1/√2[|Ψ1〉|M1〉+|Ψ2〉|M2〉]
1 x

Avatar
szabiku
Hozzászólások: 943
Csatlakozott: 2016.12.22. 01:27

Kvantummechanika Lagrange-formalizmus alapján??

Hozzászólás Szerző: szabiku » 2018.10.17. 21:53

Nem konkrétan a "kvantumradír" kísérlettel volt a baj, hanem az elnevezésével, és a szenzációhajhász tálalásával. Nincs ott semmi megjelölés meg radírozás... Teljesen félrevezető így az egész. Egyszerűen fázistolások vannak a réses ágon, és végül szelekció a koincidenciaszámlálóval, amihez a vezérlőjelet egy állítható polarizációs szűrővel választja ki a másik ágon. Ez természetesen csak összefonódottság esetén működik. Ilyen egyszerű az egész. Ezt fel is tártam a doksik hamis előadásaiból, még azzal a kezdeti tudásommal is, amit én becsületesen be is vallottam. Egyébként mikor lefordítottam szótárprogrammal a kiinduló dokumentumot (elemzés közreadva), többnyire benne is voltak azok a kijelentések, amiket én is vallottam, hogy semmi csoda nincs ott. Csak valahogy mások kapták fel a dolgot, és ferdítették, csavargatták, ontották a hülye verzióit.

Mellesleg azóta már többször is hatványozódott a tudásom. Te pedig maradtál a síkodon. :ugeek:
0 x

Avatar
Solaris
Hozzászólások: 3584
Csatlakozott: 2012.07.25. 17:32

Kvantummechanika Lagrange-formalizmus alapján??

Hozzászólás Szerző: Solaris » 2018.10.18. 06:19

szabiku írta:
2018.10.17. 21:53
Mellesleg azóta már többször is hatványozódott a tudásom.
Nem lehet, hogy a hatványkitevővel van a baj? :)
0 x

dgy
Hozzászólások: 36
Csatlakozott: 2017.01.23. 17:34

Kvantummechanika Lagrange-formalizmus alapján??

Hozzászólás Szerző: dgy » 2018.10.18. 21:03

Solaris írta:
2018.10.18. 06:19
Nem lehet, hogy a hatványkitevővel van a baj?
Nem a kitevővel, hanem az alappal. A 0,0001 kiinduló érték harmadik-negyedik hatványa elég jól leírja a helyzetet.

dgy
1 x

Avatar
szabiku
Hozzászólások: 943
Csatlakozott: 2016.12.22. 01:27

Kvantummechanika Lagrange-formalizmus alapján??

Hozzászólás Szerző: szabiku » 2018.10.20. 16:31

Úgy tűnik Gyula is egyetért a vitatott könyv 16. fejezetével. 8-)
0 x

Avatar
szabiku
Hozzászólások: 943
Csatlakozott: 2016.12.22. 01:27

Kvantummechanika Lagrange-formalizmus alapján??

Hozzászólás Szerző: szabiku » 2018.10.21. 17:13

G.Á, Gyula, api! (és hasonlók)
Látjátok már a súlyos elvi hibá(ka)t a könyvben, amire felhívtam a figyelmet??
Vagy még mindig nem?
0 x

G.Á
Hozzászólások: 92
Csatlakozott: 2017.06.23. 22:11

Kvantummechanika Lagrange-formalizmus alapján??

Hozzászólás Szerző: G.Á » 2018.10.21. 22:18

szabiku írta:
2018.10.16. 22:57
G.Á írta:
Továbbra is egyben kell nézni a kifejezést, és meglátod hogy nincs csalás.
Persze. Egyben. Meg nagyon triviálisan.
Így van.
Csak valamiért neked ez nem megy.
0 x

Avatar
szabiku
Hozzászólások: 943
Csatlakozott: 2016.12.22. 01:27

Kvantummechanika Lagrange-formalizmus alapján??

Hozzászólás Szerző: szabiku » 2018.10.22. 00:34

Lópikulát!
Na, akkor nézzük megint.
szabiku írta:
2018.10.07. 23:49
A teret a felmerülő kalkulációk során egy képletformulán belül kétszer is kell koordinátaparaméterezni. Ezt külön betűvel jelöli: x és y. (y helyett ∆x használata vacakabbá tenné a felírásokat.) Viszont az időnél nem ez a helyzet, és csak megkeverő a tx és ty használata t és ∆t helyett. Hasonlóan a térhez kettős paraméterezést használ az időre, de nem akar eltérni a t betűtől, ezért az x vagy y koordinátaparaméterezéshez tartozást indexeléssel jelöli: tx és ty. Ez persze nem csak az amatőr olvasót, hanem az amatőr szerzőt is megkeveri. Nagyon fontos, hogy tx és ty nem egy konkrét időpontot jelölnek az egy t időskálán, mint ahogy azt megszokottan gyakran ilyen indexeléssel jelöljük. A könyv szövegezése pedig pont azt sugallja, teljesen összezavar.
G.Á írta:
2018.10.15. 19:19
Egyelőre te vagy az első ember az ismeretségi körömben, akit ez a jelölés összezavar. Természetesen két konkrét időpontot jelöl, és ezek természetesen paraméterek.
:shock: :facepalm: Ez erősen azt jelenti, hogy az ismeretségi köröd gyengeelméjű.
szabiku írta:
2018.10.16. 22:57
Te most kb. azon az állásponton vagy, hogy az f(x) függvényt kifejezetten érdemes konkrétan úgy tekinteni, hogy az inkább egy c érték az xc pontban, mert f(xc) = c, és xc természetesen paraméter.
szabiku írta:
2018.10.16. 22:57
<x(tx)|y(ty)> = <x|U(tx-ty)|y> ??

U(tx-ty) vagy U(ty-tx) ??

(16.6) --> (16.13) ??
Sikerült már észrevenned, hogy az első egyenlőség baloldalán x a tx függvénye, és y a ty függvénye?
Jobboldalon <x| és |y> (egymástól és) a t időtől, tehát tx és ty mennyiségektől függetlenül báziskomponenseket jelöl. (16.20)-->(16.21)-nél ezt egyértelműen láthatod.
Továbbá G(x,tx,y,ty)-ben a zárójelben négy független változó paraméter kell szerepeljen. A (16.18) felírás egy nagy hülyeség. (16.18)-->(16.19)-ben a hülye szerző azt hiszi <x| és |y> báziskomponenseknek köze van a t indexeihez vagy értékeihez, pedig nem. A kauzalitás meg kívül áll a Schrödinger-egyenleten. A Θ(∆t) egy tégla. Egy + fedőjellel beépített hamis varázsló az egészben! (16.27) egy nagy csalás, hülyeség, már mondtam, hogy függetlenek a változók, nem egyben kell nézni azt, ahogy hibásan gondolod. Nézz utána a matematikában az LG=δ=I=1 definíciónak! Független változók vannak ott, nem olyan, hogy x meg t összefügg, t nem határozza meg x-et. A "levezetésben", melyben (16.27)-re jut, se előrefelé, se visszafelé nem állhat be ilyen marhaság.
(A 23 Útintegrálos fejezet is egy hatalmas nagy hülyeség, matematikai csalásokkal, elfedő és megtévesztő hamis magyarázásokkal... Ott is összekavarja a konkrét állapotot és a futóparaméter hely-koordinátatér báziskomponenst. A (23.2-3)-tól a (23.2) ábrával meg a (23.3) ábrával együtt hibás marhaság az egész.)
0 x

G.Á
Hozzászólások: 92
Csatlakozott: 2017.06.23. 22:11

Kvantummechanika Lagrange-formalizmus alapján??

Hozzászólás Szerző: G.Á » 2018.10.22. 09:52

:shock: :facepalm: Ez erősen azt jelenti, hogy az ismeretségi köröd gyengeelméjű.
Szerencsére nem ez az implikáció, hanem az hogy rendkívüli eréllyel szereted vitatni a definíciókat, éppúgy mint a jelölésmódokat.
Ezenkívül nem vagy tisztában konvenciók szerepével, lásd:
.

Ezek olyan hiányosságok, amelyek kezeléséhez én már nem értek.
A legelborzasztóbb, hogy valamilyen csoda folytán elfogadod
szabiku írta:
2018.10.04. 04:23
∑n[φn(x)φn*(y)] = δ(x-y)
-ot, de a sokkal triviálisabb lépést már nem. Jobb híján, próbálj meg konkrét értékeket behelyettesíteni az egyenlet két oldalára.
Ha találsz olyan értékeket ahol eltérés van, szólj.
0 x

Avatar
szabiku
Hozzászólások: 943
Csatlakozott: 2016.12.22. 01:27

Kvantummechanika Lagrange-formalizmus alapján??

Hozzászólás Szerző: szabiku » 2018.10.22. 16:22

Mi a baj a linkelt írásommal?
Csak nem azt akarod mondani, hogy a kvantummechanika úgy is felírható, hogy a pq - qp = -i helyett i ?
0 x

Avatar
szabiku
Hozzászólások: 943
Csatlakozott: 2016.12.22. 01:27

Kvantummechanika Lagrange-formalizmus alapján??

Hozzászólás Szerző: szabiku » 2018.10.22. 21:10

G.Á írta:valamilyen csoda folytán ...
Nem csoda, kérlek, hanem tudás! :) Levezetem neked:

, ahol ortonormált folytonos teljes függvényrendszer (a bázis),

és a komplex komponenssűrűség (komplex aktív-bázisvektor sűrűség :D ).

. Ezt beírva az előző kifejezésbe: , majd átrendezve:

, amiből következik, hogy:
0 x

con
Hozzászólások: 147
Csatlakozott: 2017.01.13. 12:35

Kvantummechanika Lagrange-formalizmus alapján??

Hozzászólás Szerző: con » 2018.10.22. 22:26

komplex aktív-bázisvektor sűrűség
Igen, az aktív-bázisvektor sűrűség szabiku egyik saját külön bejáratú elmeszüleménye. Ilyesmikkel bármit ki tud umbuldálni, csak az a bökkenő, hogy rajta kívül senki se ismeri a matematikának ezt a bűvészi ágát.
0 x

Avatar
szabiku
Hozzászólások: 943
Csatlakozott: 2016.12.22. 01:27

Kvantummechanika Lagrange-formalizmus alapján??

Hozzászólás Szerző: szabiku » 2018.10.22. 22:43

Az nem meglepő, hogy te nem érted, amiket előbb felírtam.
0 x

Avatar
szabiku
Hozzászólások: 943
Csatlakozott: 2016.12.22. 01:27

Kvantummechanika Lagrange-formalizmus alapján??

Hozzászólás Szerző: szabiku » 2018.10.22. 23:07

G.Á írta:de a sokkal triviálisabb lépést már nem.
(ha még nem volt elég) Akkor itt van még egy cáfolat:

Fejétől bűzlik a hal.

(16.19)-ben és (23.2)-ben ugyanazt a hülyeséget bűvöli: <x|e-iH∆t|y> , ahol azt erőlteti, hogy ∆t = tx - ty .
Ez a kifejezés erősen problémás, nem konzekvens. (A szerzőnek gőze sincs a stacionárius állapotról meg a perturbációszámításról..)
Matematikailag ez egyszerűen azt akarja jelenteni, hogy a helykoordináta-reprezentációban értett e-iH∆t operátort egyszerűen kifejtjük annak |x> , |y> báziskomponensei szerint, azaz képezzük a megfelelő mátrixelemeket. Egyébként ezeknek önmagukban egyáltalán nem is biztos, hogy van értelmes fizikai jelentésük, sőt, itt (a könyv szerinti elgondolásban) teljesen ellentmondásosak.

H = H(x,t) , azaz függ a helytől és időtől is általában. HOPPÁ!
Egy konkrét x,y paraméterhez honnan a rákból szedi a ∆t értéket? Ja persze, egyszerűen csak odaírjuk, hogy tx - ty , azt kész. A hülye amatőr olvasó ezt benyeli. Ez a matematikában nem így működik. Ráadásul kifejtés közben azért jó lenne, ha nem változna meg az a nyomorult operátor. Néhány mátrixelemmel arrébb már nem is azt az operátort fejtjük ki, mint előbb. :D Teljesen abszurd: H és ∆t függ a kifejtés paramétereitől. Jó kis dinamikus mátrixunk lesz így. Ebben még NEO is összezavarodna. :mrgreen: Pozitív vagy negatív a visszacsatolás??
0 x

con
Hozzászólások: 147
Csatlakozott: 2017.01.13. 12:35

Kvantummechanika Lagrange-formalizmus alapján??

Hozzászólás Szerző: con » 2018.10.22. 23:09

Mutasd meg Gergo73-nak!
0 x

dgy
Hozzászólások: 36
Csatlakozott: 2017.01.23. 17:34

Kvantummechanika Lagrange-formalizmus alapján??

Hozzászólás Szerző: dgy » 2018.10.22. 23:22

Csak nem azt akarod mondani, hogy a kvantummechanika úgy is felírható, hogy a pq - qp = -i helyett i ?
Hát persze hogy felírható.

Mindenki, aki öt percnél hosszabb ideig tanulta a komplex számok algebráját, tudja, hogy MINDEN képlet, amelyben i szerepel, ugyanúgy érvényes -i-vel felírva is - feltéve, ha az összes helyen konzekvensen kicseréljük az i-t az ellentettjére. Ezt a komplex algebra egy triviális automorfizmusa teszi lehetővé.

De persze van, aki soha nem tanul, csak mások kioktatásával foglalkozik. Nem olvas (vagy ha úgy csinál, mintha olvasna, akkor egyik szemén be, a másikon ki, közben az információ a mélyebb agyterületeket nem érinti), csak ír - sokat, értelmetlen szőrözéseket, felesleges akadékoskodást, agresszív pimaszkodást. Nulla tudományos tartalommal. Van ilyen. Nem kell vele foglalkozni.

dgy
1 x

Avatar
szabiku
Hozzászólások: 943
Csatlakozott: 2016.12.22. 01:27

Kvantummechanika Lagrange-formalizmus alapján??

Hozzászólás Szerző: szabiku » 2018.10.23. 00:39

Nem kell folyton ez az ócsárolgatás, úgyis tudja már mindenki...
dgy írta:Hát persze hogy felírható.

ha az összes helyen konzekvensen kicseréljük az i-t az ellentettjére. Ezt a komplex algebra egy triviális automorfizmusa teszi lehetővé.
Ezt én is tudom, de azért nem árt valami egységes konvenciót tartani, mert tarthatatlan volna, ha a szakirodalmak fele így, fele úgy építené fel az egész témáját. Éppen elég az, hogy a Fourier-transzformációnál gyakori mindkét konvenció. Egyelőre úgy tapasztaltam, hogy a kvantummechanikában a csererelációnál ez elég egységes, de találtam már eltérést. A keveredés akkor kezdődik igazán, amikor több oldalról megtett konvenciók keverednek egy témában össze egymásra rakódva. Könnyen megeshet az is, hogy ilyenkor át sem látjuk, hogy bizonyos konvenciók összefügghetnek, és hibát vétünk, ha ezeket külön akarjuk megválasztani. Ha innen-onnan veszünk képleteket, ezt mindig vizsgálgatni kell, hogy ugyanazok a konvenciók álljanak mögöttük, különben matematikai hibát vétünk. Szóval tudom mik azok a matematikai konvenciók, nem csak ezzel vagyok tisztában, hanem más jellegűekkel is, amik sokkal bonyolultabbak. Pl. a relativitáselméletben a szignatúra, stb...

De egyébként éppen te írtad egyszer, hogy: http://kozmoforum.hu/viewtopic.php?f=25 ... dott#p8635 :D
0 x

Avatar
szabiku
Hozzászólások: 943
Csatlakozott: 2016.12.22. 01:27

Kvantummechanika Lagrange-formalizmus alapján??

Hozzászólás Szerző: szabiku » 2018.10.23. 00:52

con írta:Mutasd meg Gergo73-nak!
Ez jó ötlet, lehet megkérdezem*, bár eléggé jól átlátom a dolog problematikáját, mármint, hogy pontosan mit ront el a könyv a tárgyalt részeknél. Egyértelműen látszanak a hibák, nincs kétségem.

*(A folytonos spektrumos témázgatásnál volt matematikai vaksága jócskán, amit még makacssággal is megerősített. Persze ennek ellenére igen okosnak vélem)
0 x

Avatar
szabiku
Hozzászólások: 943
Csatlakozott: 2016.12.22. 01:27

Kvantummechanika Lagrange-formalizmus alapján??

Hozzászólás Szerző: szabiku » 2018.10.23. 05:32

.
.
(16.27)---> Aki ezt nem látja, az matematikai vakságban szenved!! :mrgreen:
'
'
0 x

123
Hozzászólások: 51
Csatlakozott: 2017.02.07. 04:27

Kvantummechanika Lagrange-formalizmus alapján??

Hozzászólás Szerző: 123 » 2018.10.23. 09:44

dgy írta:
2018.10.22. 23:22
Mindenki, aki öt percnél hosszabb ideig tanulta a komplex számok algebráját, tudja, hogy MINDEN képlet, amelyben i szerepel, ugyanúgy érvényes -i-vel felírva is - feltéve, ha az összes helyen konzekvensen kicseréljük az i-t az ellentettjére. Ezt a komplex algebra egy triviális automorfizmusa teszi lehetővé.
Mármint MINDEN olyan képlet, amelyben kizárólag a "komplex algebra" műveletei szerepelnek, +-*/.

(Egyébként <R,+>-on automorfizmus a valós számok előjelének negálása, mégsem marad minden képlet érvényes, ha negálunk minden valós számot, pl az 1*1=1 sem.)
0 x

Avatar
szabiku
Hozzászólások: 943
Csatlakozott: 2016.12.22. 01:27

Kvantummechanika Lagrange-formalizmus alapján??

Hozzászólás Szerző: szabiku » 2018.10.23. 15:26

Minden művelet és összefüggés megtartódik (ami nem kifelé irányul a halmazból).
A komplex számoknál az i --> -i nem vezet ki a számhalmazból, és csak forgatási irányt vált, ezért marad minden O.K.
Amit te írsz az nem automorfizmus, nem is önmagára képez, stb...
0 x

Avatar
szabiku
Hozzászólások: 943
Csatlakozott: 2016.12.22. 01:27

Kvantummechanika Lagrange-formalizmus alapján??

Hozzászólás Szerző: szabiku » 2018.10.23. 16:02

szabiku írta:Nem kell folyton ez az ócsárolgatás, úgyis tudja már mindenki...
Ja, és az ilyenek mormolgatásától még nem lesz igazabb, ami hamis. :P
0 x

Avatar
szabiku
Hozzászólások: 943
Csatlakozott: 2016.12.22. 01:27

Kvantummechanika Lagrange-formalizmus alapján??

Hozzászólás Szerző: szabiku » 2018.10.23. 17:08

szabiku írta:A kauzalitás meg kívül áll a Schrödinger-egyenleten. A Θ(tx - ty) egy tégla. Egy + fedőjellel beépített hamis varázsló az egészben! (16.27) egy nagy csalás, hülyeség, már mondtam, hogy függetlenek a változók, nem egyben kell nézni azt, ahogy hibásan gondolod.
Teljesen kielemeztem a hibát, hatalmas! (szerintem megérdemelnék érte egy Fields-érmet! :D )

G.Á!! Nem engedlek PhD vizsgára! Nem érted sem a fizikát, sem a matematikát kellően. Alapvető dolgokon :twisted: elbuksz. elbuksz. elbuksz! Ψ
Továbbá, megfelelő erkölccsel kellene rendelkezned hozzá: aki bevallja hibáit, és megbánja bűneit. .............. Neked ebből 1-es.
0 x

G.Á
Hozzászólások: 92
Csatlakozott: 2017.06.23. 22:11

Kvantummechanika Lagrange-formalizmus alapján??

Hozzászólás Szerző: G.Á » 2018.10.23. 20:05

Teljesen kielemeztem a hibát, hatalmas!
Ja, csak a kielemzésed hibás.
szerintem megérdemelnék érte egy Fields-érmet!
Inkább bukást "matematikai módszerek a fizikában"-ból.
G.Á!! Nem engedlek PhD vizsgára!
Hát, ezzel már kissé elkéstél.
De ha komolyan szakmai/hozzáértői-tekintélyen alapuló beszélgetést szeretnél, az számodra nem éppen előnyös pozíció.
Alapvető dolgokon :twisted: elbuksz.
Pont tőled, ez kissé vicces.
Továbbá, megfelelő erkölccsel kellene rendelkezned hozzá: aki bevallja hibáit, és megbánja bűneit. .............. Neked ebből 1-es.
Én is szoktam tévedni, de veled ellentétben, nem ragaszkodom ahhoz, hogy a tévedéseim igazak.
0 x

Válasz küldése